Đến nội dung

Stranger411 nội dung

Có 85 mục bởi Stranger411 (Tìm giới hạn từ 30-03-2020)



Sắp theo                Sắp xếp  

#441489 IMO shortlist 2012 - Problems + Solution

Đã gửi bởi Stranger411 on 09-08-2013 - 15:20 trong Thi HSG Quốc gia và Quốc tế

Mới có bản Scan thôi, các bạn dùng tạm :)

File gửi kèm




#441245 Mở rộng Problem 4 - IMO 2013

Đã gửi bởi Stranger411 on 08-08-2013 - 14:40 trong Hình học

1 bài mở rộng của thầy Quang Hùng trong GGTH lần 5

Cho tam giác $ABC$, đường tròn $(I)$ đi qua $B,C$ cắt CA,AB tại $N,M$ khác $B,C$.
Đặt $H = BN \cap CM$. Gọi $d$ là đường thẳng qua $I$ vuông góc với $AH$. Lấy $W$ bất kì trên $d$.
$WK,WL$ là đường kính của đường tròn ngoại tiếp các tam giác $WBM,WCN$.
Chứng minh $K,H,L$ thẳng hàng.
 

TBr.png




#438572 Chứng minh ba đường tròn đồng quy

Đã gửi bởi Stranger411 on 27-07-2013 - 13:22 trong Hình học

Cho tứ giác toàn phần $ABCDEF$. Chứng minh các đường tròn đướng kính $AC,BD,EF$ đồng quy tại 2 điểm.

NhaSinhvien.net-bai1.png




#433417 Tìm số tự nhiên $n>4$ nhỏ nhất sa0 ch0 tồn tại $n$ ng...

Đã gửi bởi Stranger411 on 07-07-2013 - 00:26 trong Tổ hợp và rời rạc

Anh thử xem lại lời giải xem, người số 2 và 8 đâu thỏa mãn ạ. Đáp số $n=16$

khâu thử chọn anh làm sai @@!
đáp án đúng là $n=16$




#432488 Hỏi có thể thực hiện dc phép tô màu nói trên hay không nếu: 1.$n= 2012...

Đã gửi bởi Stranger411 on 03-07-2013 - 11:37 trong Tổ hợp và rời rạc

xét số nguyên $n> 1$. Người ta muốn tô màu tất cả các số tự nhiên bởi 2 màu xanh và đỏ sao cho các điều kiện sau được đồng thời thỏa mãn:

i. Mỗi số dc tô bởi 1 màu, và mỗi màu đều dc dùng để tô vô số số;

ii. tổng của n số đôi 1 khác nhau cùng màu là số có cùng màu đó.

Hỏi có thể thực hiện dc phép tô màu nói trên hay không nếu:

1.$n= 2012$

2.$n= 2013$

Tổng quát:
* Nếu $n$ lẻ thì phép tô thực hiện được:
Tô tất cả số chẵn cùng màu đỏ, số lẻ cùng màu xanh. Khi đó, mỗi số được tô bởi đúng 1 màu và có vô hạn lần tô mỗi màu (vì có vô hạn số lẻ, số chẵn). Tổng $n$ số cùng màu là một số cùng màu vì tổng $n$ số lẻ là một số lẻ (vì $n$ lẻ) và tổng $n$ số chẵn là một số chẵn.

* Nếu $n$ chẳn, thì phép tô không thực hiện được:
Phản chứng, giả sử phép tô thực hiện được, suy ra được có vô hạn số được tô bởi 2 màu xanh đỏ.
Khi đó, tồn tại số $a_1$ được tô màu xanh và $b_1 = a_1 +1$ được tô bỏi màu đỏ.

Tiếp tục, cũng tồn tại $b_2 > b_1$ mà $b_2$ được tổ bởi màu đỏ và $a_2$ được tổ bởi màu xanh.

Tương tư như vậy tồn tại $a_{n-1} > a_{n-2}$ được tô màu xanh và $b_{n-1} = a_{n-1} +1$ tô bởi màu đỏ và cũng tồn tại $b_n > b_{n-1}$ mà $b_n$ được tô màu đỏ và $a_n = b_n +1$ tô bởi màu xanh.

Tóm lại các số $a_i$ và $b_i$ là đôi một khác nhau và thỏa $a_i$ màu xanh và $b_i$ màu đỏ và $b_{2k-1} = a_{2k-1} +1$ và $b_{2k} = a_{2k} -1$ với mọi $k \le \frac{n}{2}$

Theo điều kiện đề bài thì gọi $a$ là tổng các số $a_i$ nên a được tô xanh còn $b$ là tổng các $b_i$ nên $b$ được tô đỏ. Nhưng vì $b_{2k-1} = a_{2k-1} +1$ và $b_{2k} = a_{2k} -1$ với mọi $k \le \frac{n}{2}$ nên suy ra $a=b$ từ đó $a,b$ phải được tô cùng màu dẫn tới vô lí.




#432485 Tìm số tự nhiên $n>4$ nhỏ nhất sa0 ch0 tồn tại $n$ ng...

Đã gửi bởi Stranger411 on 03-07-2013 - 11:14 trong Tổ hợp và rời rạc

Bài toán :

Tìm số tự nhiên $n>4$ nhỏ nhất sa0 ch0 tồn tại $n$ người thỏa mãn : 2 người quen nhau thì không có người quen chung còn 2 người không quen nhau thì có đúng 2 người quen chung !

Trước tiên ta chứng minh những người này có cùng người quen với nhau tại đây.
Cho mỗi người là một đỉnh của đồ thị. 2 người quen nhau thì được nối với nhau bằng 1 cạnh. Nên số người quen của mỗi người sẽ bằng với bậc của đỉnh. Suy ra đồ thì này có các đỉnh cùng bậc. Nói cách khác đây là một đồ thị đẳng cấu. Đồ thị này có $n$ đỉnh, các đỉnh có chung bậc $d$. Suy ra đồ thì có số cạnh là $m= \frac{nd}{2}$ với $d \ge 2$.

Ta tiếp tục chứng minh $|m| \leq 3(|n|-2)$.
Vì đồ thị này là liên thông nên theo định lí Euler suy ra: $|r|=|m|-|n|+2$ (r là số miền)
Gọi $n$ là tổng số cạnh thuộc các miền $r$ thì do mỗi cạnh chỉ thuộc nhiều nhất 2 miền nên $n \leq 2m$. Mỗi miền có ít nhất 3 cạnh nên $n \geq 3r$ từ đó suy ra $r \leq \frac{2}{3}m$. Thay vào công thức Euler ta đc đpcm.

Từ đó, ta có: $m= \frac{nd}{2} \le 3n-6$ $\rightarrow 12 \le (6-d)n$. Vậy $d \le 6$ (ta cần vế phải dương)

Theo các kết quả trên, ta thử chọn các trường hợp và chọn được $n_{min}=8$ với $d=3$.
Vậy $n_{min}=8$.




#432471 Topic về tổ hợp, các bài toán về tổ hợp

Đã gửi bởi Stranger411 on 03-07-2013 - 10:00 trong Tổ hợp và rời rạc

Bài 17 (cực trị-bất đẳng thức tổ hợp)

Cho tập hợp $X=\left \{ 1,2,...,50 \right \}$. Tìm số nguyên dương nhỏ nhất $k$ sao cho mọi tập con gồm $k$ phần tử của $X$ đều chứa hai phần tử phân biệt $a$ và $b$ sao cho $ab$ chia hết cho $a+b$

Ta có các cặp số thoả mãn $ab$ chia hết $a+b$: $(3;6);(4;12);(5;20);(6;12);(6;30);(7;42);(8;2);(9;18);(10;15);(10;40);(12;24);(12;36);(14;35);(15;30);(16;48);(18;36);(20;30);(21;28);(21;42);(24;40);(24;48);(30;45);(36;45)$

Tổng hợp các số lại tập $A$, ta có:\[A = \left\{ {3;4;5;6;7;8;9;10;12;14;15;16;18;20;21;24;30;35;36;40;45;48} \right\}\]

\[ \Rightarrow k \ge \left| {X\backslash A} \right| + \left[ {\frac{{\left| A \right|}}{2}} \right] + 1 = 39\]

Vậy $min(k)=39$

--------------------------------------------------------------------------------------------------------------------------------

Đây là kết quả sau 1h bấm máy tính của mình, các bạn có cách nào hay hơn thì đưa lên để mọi người cùng tham khảo

Giải bài 17:
Chứng minh $k \ge 39$
Các cặp số thoả mãn $ab$ chia hết $a+b$: $(3;6);(4;12);(5;20);(6;12);(6;30);(7;42);(8;2);(9;18);(10;15);(10;40);(12;24);(12;36);(14;35);(15;30);(16;48);(18;36);(20;30);(21;28);(21;42);(24;40);(24;48);(30;45);(36;45)$

Xét tập $M=(6,12,15,18,20,21,24,35,40,42,45,48)$. Vì 23 cặp trên đều có phần tử thuộc $M$ nên tập $X\M$ không thỏa mãn bài toán. Mà $|X\M|=38 \rightarrow k \ge 39$

Chứng minh mọi tập có 39 phần tử đều thỏa mãn bài toán:
Xét tập $A$ bất kì gồm 39 phần tử của $X$.
Chọn 12 cặp số trong 23 cặp trên sao cho các phần tử không trùng nhau: $(3;6);(4;12);(5;20);(7;42);(8;2);(9;18);(10;15);(14;35);(18;36);(21;28);(24;40);(30;45)$

12 cặp trên chứa 24 phần tử của $X$. Nên $X$ chỉ còn lại 26 phần tử.
Vậy ít nhất 13 phần tử của $A$ của phải thuộc 12 cặp trên.
Theo nguyên lí drichlet thì $A$ chứa ít nhất 1 trong 12 cặp trên.
Từ đó tập $A$ thỏa mãn đề bài.




#432263 Topic về tổ hợp, các bài toán về tổ hợp

Đã gửi bởi Stranger411 on 02-07-2013 - 12:05 trong Tổ hợp và rời rạc

Post tiếp 2 bài nữa làm ... điểm tâm  :wacko: :

Bài 15:(APMO 1998) Cho F là tập tất cả các bộ (A1, . . . ,An) sao cho mỗi Ai là một tập con của {1, 2, . . . , 1998}. Ký hiệu |A| là số các phần tử của tập hợp A. Hãy tính

$\sum\limits_{({A_1},{A_2},...,{A_n}) \in F} {|{A_1} \cup {A_2} \cup ... \cup {A_n}|} $

Bài 16:(Bulgaria 1996) HÌnh chữ nhật $m \times n ( m,n \in N, m,n>1)$ được chia thành $mn$ hình vuông đơn vị. Có bao nhiêu cách xoá 2 hình vuông sao cho phần còn lại có thể lát kín bởi các domino?

Tại sao các bạn cứ thích post những bài không có lời giải ở trong sách ra thế nhỉ :))
Mình nhớ không nhầm thì mấy bài này trong chuyên đề của thầy Huỳnh Tấn Châu để học sinh tự giải. Sau đây là cách của mình, bạn có cách khác thì post lên cho mọi người tham khảo.
Giải bài 15:

Bài này đếm bằng truy hồi.
Tập $(1, 2, . . . , 1998)$ có tất cả $2^{1998}$ tập con $A_i$ nên tổng cần tính có tất cả $2^{1998n}$ bộ $n$ số.
Bước 1: Với các tập con $(A_1, A_2 ... , A_n)$ của $(1, 2, . . . , 1997)$, ta có thể thêm hoặc không thêm vào mỗi tập $A_i$ phần tử $1998$ để tạo thành tập con của tập $(1, 2, . . . , 1998)$. Vậy từ bộ $(A_1, A_2 ... , A_n)$ ta có $2^n$ bộ gồm các tập con của $\left \{1, 2, . . . , 1998 \right \}$.

Bước 2: Bây h chỉ còn xét các tập con $(A_1, A_2 ... , A_n)$ của các bộ còn lại. Làm tương tự như trên thì ta có được $(2^n -1).2^{n(m-1)}$.
Tình tổng lại, ta có:

$\sum\limits_{({A_1},{A_2},...,{A_n}) \in (1,...,1998)} {|{A_1} \cup {A_2} \cup ... \cup {A_n}|} = (2^n.\sum\limits_{({A_1},{A_2},...,{A_n}) \in (1,...,1998)} {|{A_1} \cup {A_2} \cup ... \cup {A_n}|}) + (2^n -1) 2^{n(m-1)}$
Từ đó, ta có: $\sum\limits_{({A_1},{A_2},...,{A_n}) \in F} {|{A_1} \cup {A_2} \cup ... \cup {A_n}|} = 1998(2^{1998n} - 2^{1997n})$.

 

Mở rộng bài 15: Tính:
$P= \sum_{(A_{1},A_{2},...,A_{k})\in F_{k}}\sum_{b\in (A_{1}\cup A_{2}\cup ...\cup A_{k})}b$
$S= \sum_{(A_{1},A_{2},...,A_{k})\in F_{k}}\sum_{i=1}^{k}\sum_{b\in A_{i}}b$
 

Bài 7: (Đề đề nghị 30/4 -2012) Trong một kì thi hoa hậu, mỗi thành viên của ban giám khảo được quyền đề xuất $10$ người đẹp vào vòng chung kết. Một nhóm người đẹp được gọi là nhóm ưng ý đối với giám khảo A nếu trong nhóm đó có ít nhất một người đẹp mà A đề xuất. Biết rằng với 6 giám khảo bất kỳ luôn tồn tại một nhóm gồm 2 người đẹp là nhóm ưng ý đối với mỗi giám khảo trong 6 giám khảo đó. Chứng minh rằng tồn tại một nhóm gồm 10 người đẹp là nhóm ưng ý đối với mỗi thành viên của ban giám khảo.

Đề này của trường chuyên Nguyễn Tất Thành - Komtum và họ không đưa đáp án nên trong sách không có lời giải.
Mình nghĩ bạn Ispectorgadget cũng không có đáp án cho bài này. (Trừ khi bạn ấy học trường chuyên NTT)

Bài này chắc không thể giải bằng đồ thị vì phải xét tới 2 đối tượng là hoa hậu và giám khảo.
Thôi thì phát biểu lại dưới dạng tập hợp để mọi người cùng nghiên cứu:
Cho $X=\{1;2;...;n\}$. Và $A_1 ,A_2, ..., A_m$ là các tập con của $X$.
Biết 6 phần tử bất kì của $X$ luôn thuộc $|A_i\cup A_j|$ nào đó.
Chứng minh tồn tại $i_1;i_2;..;i_{10}$ phân biệt mà $|\bigcup\limits_{j=1}^{10}A_{i_j}|=X$.




#431947 Topic về tổ hợp, các bài toán về tổ hợp

Đã gửi bởi Stranger411 on 30-06-2013 - 22:38 trong Tổ hợp và rời rạc

Bài 11:(Lý thuyết đồ thị, APMO 1989) Chứng minh rằng một đồ thị n đỉnh, k cạnh thì sẽ có ít nhất $\frac{{k\left( {4k - {n^2}} \right)}}{{3n}}$ tam giác.

Câu này được chế biến lại làm đề chọn đội tuyển của KHTN năm nào đó thì phải, nhưng cũng chỉ là điều kiện cần.
Giải bài 11:
Gọi $D_i$ là bậc của đỉnh $i$ nào đó.
Nếu 2 đỉnh $i$ và $j$ liên thông thì tồn tại ít nhất $D_i + D_j - 2$ cạnh khác nổi đến $n-2$ đỉnh khác của đồ thị.
Nên có $D_i + D_j - n$ tam giác có đỉnh là $j$ và $j$. Vậy số tam giác có chứa cạnh $ij$ được tạo thành phải tối thiểu là:
$\sum_{i,j} {\frac{D_i + D_j -n}{3}}=\sum_{i,j} {\frac{D_i + D_j}{3} - \frac{nk}{3}} = \sum_{i} {\frac{D_i^2}{3} - \frac{nk}{3}}$

Vậy số tam giác tối thiểu được tạo thành là:

$\sum_{i}{\frac{D_i^2}{3}} -\frac{nk}{3} \geq \frac{1}{3n}\left ( \sum_{i}{D_i} \right )^2 - \frac{nk}{3} = \frac{4k^2}{3n} -\frac{nk}{3}$
Q.E.D.

 

Bài 6: (truy hồi/đa thức,giải tích tổ hợp,PTNK 2009) Cho số nguyên dương n. Có bao nhiêu số chia hết cho 3, có n chữ số và các chữ số đều thuộc {3, 4, 5, 6}?

Sau đây là cách giải PP truy hồi:

Gọi $a_n$ là số các số có $n$ chữ số lập từ ${3, 4, 5, 6}$ và chia hết cho 3, còn $b_n$ là số các số có $n$ chữ số lập từ ${3, 4, 5, 6}$ và không chia hết cho 3. Khi đó ta có

$a_n = 2a_{n-1} + b_{n-1}(1)$

$b_n = 2a_{n-1} + 3b_{n-1}(2)$

Từ (1) suy ra $b_{n-1} = a_n – 2a_{n-1}$, thay n à n+1 thì được $b_n = a_{n+1} – 2a_n$. Thay vào (2), ta được

$a_{n+1} – 2a_n = 2a_{n-1} + 3(a_n – 2a_{n-1})$

$a_{n+1} – 5a_n + 4a_{n-1} = 0$.

Giải phương trình sai phân này, với chú ý rằng a1 = 2, a2 = 6, ta tìm được

\[{a_n} = \frac{{{4^n} + 2}}{3}.\]

Với cách đặt trên ta có thể có công thức truy hồi khác là $b_n = 4^n - a_n$
từ đó suy ra $a_{n+1} = 2a_{n} + b_{n}= a_n + 4^n$
Nên $a_{n} = 4^{n-1} + 4^{n-2} +...+ a_{1} = \frac{4^n + 2}{3}$




#431535 Topic về tổ hợp, các bài toán về tổ hợp

Đã gửi bởi Stranger411 on 29-06-2013 - 11:02 trong Tổ hợp và rời rạc

Bài 2: (PP ánh xạ, chứng minh đặc tính tổ hợp) Gọi an là số các xâu nhị phân độ dài n không chứa chuỗi con 010, bn là số các xâu nhị phân độ dài n không chứa chuỗi con 0011 hoặc 1100. Chứng minh rằng bn+1 = 2an với mọi n nguyên dương. 

Giải bài 2:
Xét một xâu nhị phân bất kì  $\left \{ x_1, x_2,...,x_n \right \}$.
Ta xây dựng một xâu nhị phân $\left \{ y_1, y_2,...,y_{n+1} \right \}$ sao cho $y_1 =0$ và $y_k = x_1 + x_2 +...+ x_k(mod2)$
Rõ ràng, xâu $\left \{ x_1, x_2,...,x_n \right \}$ có dạng $a_n$ khi và chỉ khi $\left \{ y_1, y_2,...,y_{n+1} \right \}$ có dạng $b_n$. Nói cách khác đó là một song ánh biến các xâu có dạng $a_n$ thành các xâu có dạng $b_{n+1}$ bắt đầu bằng $0$.
Với mỗi xâu $\left \{ y_1, y_2,...,y_{n+1} \right \}$, ta thay các kí tự 1 bởi 0 và 0 bởi 1, ta được một xâu khác cũng có dạng $b_{n+1}$ nhưng bắt đầu bởi $1$.
Từ đó cho ta: $b_{n+1} = 2a_n$.
 

Bài 3 (Toán trò chơi-Nguồn: Tournament of the Towns). Trên bảng ô vuông $20$x$20$ mỗi ô có $1$ quân cờ. $A$ chọn một số thực $d$ và $B$ phải đưa mỗi quân cờ đi tới ô cách ô ban đầu nó đứng một khoảng cách ít nhất là $d$ (khoảng cách giữa $2$ ô được tính theo khoảng cách tâm của $2$ ô đó) và mỗi ô chỉ có thể có $1$ quân cờ. Hỏi với điều kiện gì của $d$ thì $B$ có thể thao tác thoả mãn điều kiện trên?

Hỏi tương tự với bảng $21$x$21$

Với bài này có lẽ ta phải chia bàn cờ ra 4 phần.
Cái đáp số hơi lằng nhằng, không biết có đúng không.




#430322 Đề thi vào lớp 10 THPT chuyên Lê Quí Đôn Đà Nẵng 2013-2014 (Hệ số 2)

Đã gửi bởi Stranger411 on 24-06-2013 - 20:42 trong Tài liệu - Đề thi

 

Bài 4. (1,5 điểm)

          Cho một tháp số (gồm 20 ô vuông giống nhau) như hình vẽ. Mỗi ô vuông được ghi một số nguyên dương n với $1\leq n\leq 20$, hai ô vuông bất kỳ không được ghi cùng một số. Ta quy định trong tháp số này 2 ô vuông kề nhau là 2 ô vuông có chung cạnh. Hỏi có thể có cách ghi nào thỏa mãn điều kiện: Chọn 1 ô vuông bất kỳ (khác với các ô vuông được đặt tên a, b, c, d, e, f, g, h như hình vẽ) thì tổng của số được ghi trong ô đó và các số được ghi trong 3 ô vuông kề với nó chia hết cho 4 ?

1001458_10200372004477893_499076836_n.jp




#423590 Chứng minh 3 điểm cùng thuộc một đường vuông góc với $OE'$

Đã gửi bởi Stranger411 on 03-06-2013 - 23:15 trong Hình học

Cho tam giác $ABC$ có $O$ là tâm đường tròn ngoại tiếp, $E$ là tâm đường tròn Euler. Lấy $E'$ thỏa $\widehat{E'BA}=\widehat{EBC}$ và $\widehat{E'AB}=\widehat{EAC}$. Trung trực $OA$ cắt $BC$ tại $A'$. Các điểm $B',C'$ được xác định tương tự. Chứng minh $A',B',C'$ cùng thuộc 1 đường thẳng vuông góc với $OE'$

 

 

 

 




#422851 Chứng minh tiếp tuyến

Đã gửi bởi Stranger411 on 01-06-2013 - 17:28 trong Hình học

Qua $M$ vẽ đường thẳng song song với $d$,Cắt $(O)$ tại $N$,cắt $OH$ tại $I$.

 

Vì $HA=HB$ và $MN//AB$ nên $M(AB,HN)=-1$

Theo mình thấy chỗ này nên chứng minh lại bổ đề sau:

 

Bổ đề 1: Cho $a,b,c,d$ là chùm đường thẳng tâm $O$. Đường thẳng $\delta$ không đi qua $O$, cắt $a,b,c,d$ tại $A,B,C,D$. Đường thẳng $\delta'$ không đi qua $O$, cắt $a,b,c$ tại $A',B',C'$. Khi đó $\delta' // d$ thì $(ABCD)=(A'B'C')$.
 

Áp dụng bổ đề trên thì $M(ABHN)= (ABH) = -1$.




#422841 Chứng minh tiếp tuyến

Đã gửi bởi Stranger411 on 01-06-2013 - 16:36 trong Hình học

Bài toán khá hay đó bạn:

Qua $M$ vẽ đường thẳng song song với $d$,Cắt $(O)$ tại $N$,cắt $OH$ tại $I$.

 

Vì $HA=HB$ và $MN//AB$ nên $M(AB,HN)=-1$,Chiếu chùm điều hòa lên $(O)$ ta có tứ giác $DCEN$

 

Kí hiệu mà bạn dùng chỗ nào là thế nào vậy ?
mình xem lại lí thuyết về chùm điều hòa mà không thấy.




#422825 Chứng minh tiếp tuyến

Đã gửi bởi Stranger411 on 01-06-2013 - 15:17 trong Hình học

Cho đường tròn $(O)$ và đường thẳng $d$. Gọi $H$ là hình chiếu của $O$ lên $d$. Cho 2 điểm $A,B \in d$ sao cho $HA =HB$. Lấy $M$ bất kì trên $(O)$. $MH,MA,MB$ lần lượt cắt $(O)$ tại $C,D,E$. Gọi $S$ là giao điểm của d và $DE$. Chứng minh $SC$ là tiếp tuyến của đường tròn $(O)$.




#411157 $a^7 + 7 = b^2$

Đã gửi bởi Stranger411 on 07-04-2013 - 21:14 trong Số học

Tìm tất cả các cặp số nguyên dương $(a,b)$ thỏa mãn:
$$a^7+7=b^2$$




#411014 $$\sum^n_{k=0}2^k \binom{n}{k...

Đã gửi bởi Stranger411 on 07-04-2013 - 12:06 trong Các dạng toán khác

hứng minh:
$$\sum^n_{k=0}2^k \binom{n}{k} \binom{n-k}{\left [ \frac{n-k}{2} \right ]}=\binom{2n}{n}$$

 

@Dark templar: Bài này thấy quen quá,hình như đã có trong ĐTTH,phần Hàm sinh...


Do cái vế phải hình như bị sai thôi anh dark templar à zz


Đảo chiều 1 phát, ta được:

$$\sum\limits_{k = 0}^n {{2^k}\binom{n}{k}\binom{n - k}{\left\lfloor {\frac{{n - k}}{2}} \right\rfloor }}  = \sum\limits_{k = 0}^n {{2^{n - k}}\binom{n}{k}\binom{k}{\left\lfloor {\frac{k}{2}} \right\rfloor }} $$

Vì $\binom{k}{\left\lfloor \frac{k}{2} \right\rfloor}$ là hạng tử tự do trong khai triển $(1+x)\left(x+\frac{1}{x} \right)^{k}$.
Từ đó, ta có:

$$\sum^n_{k=0}2^{n-k} \binom{n}{k} (1+x)\left ( x+ \frac{1}{x} \right )^k = (1+x)\left ( 2+x+ \frac{1}{x} \right )^n= \frac{1}{x^n} (1+x)^{2n+1}$$
Xét hạng tử tự do ở 2 vế, ta được:

$$\sum^n_{k=0}2^k \binom{n}{k} \binom{n-k}{\left [ \frac{n-k}{2} \right ]}=\binom{2n+1}{n}$$



#410910 CMR luôn tồn tại hai số $k$ khác $m$ sao cho: $|a_m-...

Đã gửi bởi Stranger411 on 06-04-2013 - 22:08 trong Tổ hợp và rời rạc

Cho các số dương $a_1;a_2;...;a_7$, $b_1;b_2;...;b_7$ thỏa: $a_i+b_i \le 2$. CMR luôn tồn tại hai số $k$ khác $m$ sao cho: $$|a_m-a_k|+|b_m-b_k| \le 1$$.

 

@Reddevil: Dùng ý tưởng hình học cho mấy bài này thường không được tự nhiên cho lắm zz 

Xét giao của các tập sau với tập $\{(x,y)| x\ge 0, y\ge 0, x+y \leq 2\}$:
$|x-\frac{1}{2}|+|y| \leq \frac{1}{2}$, 
$|x|+|y-\frac{1}{2}| \leq \frac{1}{2}$,
$|x-\frac{3}{2}|+|y| \leq \frac{1}{2}$,
$|x|+|y-\frac{3}{2}| \leq \frac{1}{2}$,
$|x-\frac{1}{2}|+|y-1| \leq \frac{1}{2}$,
$|x-1|+|y-\frac{1}{2}| \leq \frac{1}{2}$.
6 tập này phủ toàn bộ $\{(x,y)| x\ge 0, y\ge 0, x+y \leq 2\}$. 
Như vậy tồn tại ít nhất một tập chứa hai cặp $(a_i,b_i)$ và $(a_j,b_j)$ nào đó.
Khi đó $|a_i-a_j|+|b_i-b_j|\leq 1$.



#409245 Tìm số dư của phép chia $S=\prod^{p}_{t=1}(t^...

Đã gửi bởi Stranger411 on 30-03-2013 - 22:03 trong Số học

Bài toán:

Ch0 số nguyên tố lẻ $p=mk+2$ tr0ng đó $m.k\in \mathbb{N}^{*},m>2$. Tìm số dư của phép chia $S=\prod^{p}_{t=1}(t^{m-1}+t^{m-2}+...+t+1)$ ch0 $p$.

Một bài khó đánh giá hơn ;)

Cho số nguyên tố $p \equiv 1( \bmod m)$, $m  >2$. Chứng minh:
\[\prod\limits_{t = 1}^p {\left( {{t^{m - 1}} + {t^{m - 2}} +  \ldots  + 1} \right)}  \equiv 0(\bmod p)\]




#400670 Tô màu các số tự nhiên

Đã gửi bởi Stranger411 on 28-02-2013 - 14:40 trong Tổ hợp và rời rạc

Nếu $n$ là số lẻ thì hoàn toàn thực hiện được. Tô tất cả số chẵn cùng màu đỏ, số lẻ cùng màu xanh. Khi đó, cách tô trên là thỏa vì:
(i) Mỗi số được tô bởi đúng 1 màu và có vô hạn lần tô mỗi màu (vì có vô hạn số lẻ, số chẵn)
(ii) Tổng $n$ số cùng màu là một số cùng màu vì tổng $n$ số lẻ là một số lẻ (vì $n$ lẻ) và tổng $n$ số chẵn là một số chẵn.
======================================
Nhưng nếu $n$ chẵn thì "có thể" không tô được. Nhưng không biết chứng minh sao :))

Cái trường hợp $n$ chẳn của bạn chỉ là điều kiện cần thôi :lol:
Bài này dù $n$ chẳn hay $n$ lẻ đều không tô được :)

Spoiler



#400613 Tô màu các số tự nhiên

Đã gửi bởi Stranger411 on 28-02-2013 - 09:13 trong Tổ hợp và rời rạc

Cho số tự nhiên $n>1$. Người ta tô màu tất cả các số tự nhiên bằng 2 màu xanh và đỏ thỏa mãn 2 đk:
1) Mỗi số chỉ được tô một màu và mỗi màu được dùng để tô vô hạn số.
2) Tổng của $n$ số phân biệt cùng màu, là một số cũng có màu đó.
Hỏi cách tô trên có thể thực hiện được hay không ?

Spoiler



#398293 Những người phát cuồng vì tramyvodoi

Đã gửi bởi Stranger411 on 19-02-2013 - 19:01 trong Góc giao lưu

Post cái hình lên :v
Bạn nào thích thì cứ lấy làm ava lun nhá :-j

Hình đã gửi



#397889 Những người phát cuồng vì tramyvodoi

Đã gửi bởi Stranger411 on 17-02-2013 - 23:58 trong Góc giao lưu

Các cháu trên faceboook đang phát cuồng :D
Mong bạn tramyvodoi đừng giận nhá :))
Hình đã gửi

Và đây là hậu quả của những cháu đua đòi :-ss
Hình đã gửi



#395098 $\sum\limits_{k=0}^{n}\binom{x+1...

Đã gửi bởi Stranger411 on 09-02-2013 - 00:29 trong Tổ hợp và rời rạc

Ta có định lý B như sau:
\begin{equation}
\sum\limits_k {{n+ak \choose m+bk}} f_k = \left[ {t^m } \right]\left( {1 + t} \right)^n f\left( {t^{ - b} \left( {1 + t} \right)^a } \right) \quad (b<0)
\end{equation}
====================================
Quay lại bài toán.Áp dụng định lý B cho VT, ta có:
$$\begin{array}{rcl} \sum\limits_{k = 0}^n \binom{x - 2k}{n - k}\binom{x + 1}{2k + 1}2^{2k + 1} &=& \left[ {t^n } \right]\left( {1 + t} \right)^x \left[ {\frac{{\left( {1 + 2\sqrt u } \right)^{x + 1} - \left( {1 - 2\sqrt u } \right)^{x + 1} }}{2\sqrt u} \bigg| u = \frac{t}{{\left( {t + 1} \right)^2 }}} \right]\\
&=& \left[ {t^n } \right]\left( {1 + t} \right)^x \left[ {\frac{{\left( {1 + 2\sqrt t + t} \right)^{x + 1} - \left( {1 - 2\sqrt t + t} \right)^{x + 1} }}{{2\dfrac{\sqrt t}{t+1}\left( {1 + t} \right)^{x + 1} }}} \right]\\
&=& \left[ {t^n } \right]\frac{{\left( {1 + \sqrt t } \right)^{2x + 2} - \left( {1 - \sqrt t } \right)^{2x + 2} }}{2\sqrt t}\\
\text{Mà ta có: }& & \\
\dfrac{\left( {1 + \sqrt t } \right)^{2x + 2} - \left( {1 - \sqrt t } \right)^{2x + 2}}{2\sqrt t} &=& \dfrac{1}{2\sqrt t}\left( {\sum\limits_{k = 0}^\infty {\binom{2x + 2}{k}\left( {\sqrt t } \right)^k } - \sum\limits_{k = 0}^\infty {\binom{2x + 2}{k}\left( { - \sqrt t } \right)^k } } \right)\\
&=& \dfrac{1}{2\sqrt t}.2\sum\limits_{k = 0}^\infty {\binom{2x + 2}{2k + 1}\left( {\sqrt t } \right)^{2k + 1} }\\
&=& \sum\limits_{k = 0}^\infty {\binom{2x + 2}{ 2k + 1}t^k }\\
\Rightarrow \left[ {t^n } \right]\dfrac{\left( {1 + \sqrt t } \right)^{2x + 2} - \left( {1 - \sqrt t } \right)^{2x + 2}}{2\sqrt t } &=& \binom{2x + 2}{2n + 1}\\
\Rightarrow \sum\limits_{k = 0}^n {\binom{x - 2k}{n - k}\binom{x + 1}{2k + 1}2^{2k + 1}} &=&\binom{2x + 2}{2n + 1}
\end{array}$$

Em cũng ko ngờ là lại có một lời giải như thế này.
Đây là bài toán được sáng tạo từ công thức "Leibniz suy rộng".
Không biết cái công thức trên thì khó mà xây dựng cái hàm sinh cho bài này.

Xin trình bày lời giải của 1 chú lớp 11 cùng trường vs Perfectstrong ;)
Xét hàm: $$f(y)=\sum\limits_{k=0}^{n} \binom{x+1}{t}2^ty^t(1+y^2)^{x+1-t},\left | y \right | < 1$$
Bằng cách thay $t=2k+1$, ta được hệ số của $y^{2n+1}$ là:
$$\sum\limits_{k=0}^{n}\binom{x+1}{2k+1}\binom{x-2k}{n-k}2^{2k+1}$$
Mà $f(y)= (2y+ (1+y^2))^{x+1} = (1+y)^{2x+2}$, nên hệ số của $y^{2n+1}$ trong khai triển này là: $\binom{2x+2}{2n+1} $
Q.E.D

@supermember: Anh chưa xem kĩ nhưng lời giải này không biết có đúng không nha :), cái $x$ trong đề không ghi nhưng nó là số thực chứ không phải số tự nhiên, như vậy thì cái công thức nhị thức đưa ra trong lời giải này hơi hên xui àh nha :)

@Stranger411: Theo định nghĩa hệ số nhị thức mở rộng thì
Với u là số thực, k là số nguyên không âm thì

\[\left( \begin{array}{l}
u\\
k
\end{array} \right) = \left\{ \begin{array}{l}
\frac{{u(u - 1) \ldots (u - k + 1)}}{{k!}}(k > 0)\\
1(k = 0)
\end{array} \right.\]
Nên em nghĩ lời giải trên chắc không có vấn đề.



#393747 $\sum\limits_{k=0}^{n}\binom{x+1...

Đã gửi bởi Stranger411 on 06-02-2013 - 13:39 trong Tổ hợp và rời rạc

Chứng minh:
$$\sum\limits_{k=0}^{n}\binom{x+1}{2k+1}\binom{x-2k}{n-k}2^{2k+1}=\binom{2x+2}{2n+1} \forall x \in \mathbb{R}$$